Módulo 2 Psiquiatra Flashcards

(50 cards)

1
Q

A 35-year-old married man presents to his primary care physician 1 month after a recent hospitalization for a car accident, which left him with limited use of his arm and hand. The patient, who is a local contractor building houses, was instructed to avoid any gripping, arm raising, and other strenuous activities that his work had previously required. He continues to oversee his workers on the job but now complains that he finds himself anxious and despondent at times, with episodes of irritability and tearfulness and a decreased interest in his work. He still enjoys coaching his son’s baseball team, and he has had no sleep or appetite changes. He also feels that his sexual desire has diminished. The patient has no prior personal or familial history of a mood disorder or psychiatric disorder. Which of the following is the most likely diagnosis?

(A) Adjustment disorder with depressed mood
(B) Bipolar disorder, most recent episode depressed
(C) Conversion disorder
(D) Dysthymic disorder
(E) Major depressive disorder

A

Respuesta: A

The correct answer is A. The best diagnosis is adjustment disorder with depressed mood, as the patient had several stressors including a recent hospitalization, a debilitating injury, and an inability to perform his job duties as he had done previously. As a result of these changes, the patient developed some of the symptoms of depression without a full set of symptoms, such as insomnia or anorexia.

The patient does not have any history of manic episodes, which means bipolar disorder, most recent episode depressed (choice B), is not a possibility.

Conversion disorder (choice C) is a psychiatric disorder in which one or more physical symptoms or deficits affect voluntary motor or sensory function. The diagnosis requires that a symptom or deficit cannot be fully explained by a general medical condition, which is not the case for this patient.

Dysthymic disorder (choice D) is a disorder in which mood is depressed for most of the day, for more days than not, for at least 2 years. This diagnosis is not a possibility because the patient’s depressive symptoms are limited to this episode only.

Major depressive disorder (choice E) is not the appropriate diagnosis, as his symptoms have not lasted for more than a few weeks, and the patient does not have the full vegetative set of symptoms of major depressive disorder.

How well did you know this?
1
Not at all
2
3
4
5
Perfectly
2
Q

A 57-year-old Asian American man, accompanied by his wife, presents to a psychiatrist. The patient has no significant past medical or psychiatric history. He admits to depressed mood, decreased appetite, a 10-pound weight loss, initial insomnia, decreased ability to concentrate, and mild memory problems for the past 3 weeks. On further questioning, he states that he feels hopeless and helpless. On mental status examination, the patient appears calm and is cooperative. He has a sad affect. There is no evidence of a thought disorder. Which of the following would be the most appropriate next step in management?

(A) Assessment of suicidality
(B) Cognitive-behavioral therapy
(C) Initiation of a selective serotonin reuptake inhibitor
(D) Initiation of a benzodiazepine
(E) Urine drug screen

A

Respuesta: A

The correct answer is A. The patient has symptoms suggestive of major depressive disorder, and assessment of suicide risk is paramount in interviewing him. Depressed patients have a significantly higher risk of attempting and completing suicide compared with the general population. Therefore, determining whether a depressed patient is at high risk is essential early in their management.

Although cognitive-behavioral therapy (choice B) may be an important adjunct in treating depression, it is not the most appropriate next step in this patient’s management.

Although a selective serotonin reuptake inhibitor (SSRI) (choice C) would be an acceptable pharmacologic intervention in this patient with depression, assessing suicidality takes precedence.

A benzodiazepine (choice D) may be acceptable on a short-term basis in an individual who is acutely agitated or has significant symptoms of anxiety. This patient, however, appears calm and cooperates with the examiner, and there is no suggestion for the need for acute use of a benzodiazepine.

A urine drug screen (choice E) may prove helpful in differentiating a major depressive episode from a substance-induced mood disorder. However, even patients with a substance-induced mood disorder are at higher risk for suicide than the general population, and assessment of suicidality must occur early in management.

How well did you know this?
1
Not at all
2
3
4
5
Perfectly
3
Q

A 47-year-old Asian American man visits his physician for problems falling asleep. The man says he goes to bed every night at 11 PM and arises at 6 AM; however, he does not fall asleep until about 60 minutes after going to bed. He denies depressed mood or recent psychosocial stressors. When asked about his daily routine, he reports that he takes a brisk walk every morning and then goes to work. On the way to work, he drinks one cup of coffee, but denies any other caffeine intake during the remainder of the day. At around 9:30 PM, he eats dinner and then watches television until 11 PM, at which time he drinks 1 ounce of whiskey before retiring. Which of the following would be the best advice for this patient?

(A) Decrease the intensity of the morning exercise
(B) Eat dinner earlier in the evening
(C) Exercise vigorously just prior to going to bed
(D) Increase the amount of alcohol drunk before retiring
(E) Use zolpidem at bedtime as needed

A

Respuesta: B

The correct answer is B. The first step in evaluating insomnia in a patient, after ruling out a mood disorder, is to review the “sleep hygiene” activities that affect the sleep-wake cycle, such as going to bed and waking up at the same time every day. In this case, eating a large meal before retiring might be the reason why this patient is having difficulty falling asleep. Encouraging him to eat an earlier dinner would be an appropriate adjustment in his routine.

Increasing the intensity of the morning workout would likely improve his sleep; decreasing it would prove unhelpful (choice A).

Vigorous exercise just prior to going to bed (choice C) would likely exacerbate the insomnia.

Although a nightcap (choice D) may help the patient fall asleep initially, its effect on sleep architecture would likely keep him from feeling rested on awakening. Increasing his alcohol intake prior to retiring will probably exacerbate this problem.

The use of medication, such as zolpidem (choice E), is reserved for situations in which adjustments in sleep hygiene prove ineffective. In addition, hypnotics are intended to be used on a short-term basis only.

How well did you know this?
1
Not at all
2
3
4
5
Perfectly
4
Q

A 15-year-old high school sophomore is brought to the emergency department by his father because of bizarre behavior during the past week. Two weeks earlier, the patient’s twin brother was killed in a car accident. After the funeral, while helping to go through his brother’s things, the boy began screaming and flailing about wildly, saying that his brother’s guardian angel was coming to get him for not taking better care of his brother. The patient had been an excellent student and a popular athlete, with no previous psychiatric history. Which of the following is the most likely diagnosis?

(A) Brief psychotic disorder
(B) Delusional disorder
(C) Drug intoxication
(D) Grief reaction
(E) Schizophrenia

A

Respuesta: A

The correct answer is A. Brief psychotic disorder is a diagnosis that requires the sudden onset of a florid psychotic episode immediately after a marked psychosocial stressor in the absence of increasing psychopathology before the stressor.

Delusional disorder (choice B) is incorrect because this disorder requires nonbizarre delusions of at least 1 month’s duration, which has not occurred in this case.

Drug intoxication (choice C) often can present with symptoms similar to brief psychotic disorder, but there is no indication of substance use in this case.

Grief reaction (choice D) is an expected and normal reaction to the loss of a loved one. The patient’s behavior and psychosis are not within the realm of a normal reaction.

Schizophrenia (choice E) requires symptoms of psychosis for at least 6 months.

How well did you know this?
1
Not at all
2
3
4
5
Perfectly
5
Q

A 62-year-old Iranian American man is seen for the first time by a physician for complaints of abdominal pain. The patient states that the pain has been present for more than a year, is localized to the lower abdomen, and is “sharp and crampy” in character. He has brought with him a rather sizable stack of medical records. A careful review of the records reveals that the patient has had the same complaint for approximately 3 years, and several extensive and redundant evaluations by various physicians for pathology have been uninformative. When the physician asks the patient what he believes to be responsible for the pain, he replies, “I don’t believe it’s anything-know it’s cancer.” When confronted with the contents of his medical records, which reveal no malignancy, the patient states, “Colon cancer runs in my family. I know that if you run another test, it’ll show something serious.” Which of the following is the most likely diagnosis?

(A) Body dysmorphic disorder
(B) Conversion disorder
(C) Hypochondriasis
(D) Schizophrenia
(E) Somatization disorder

A

Respuesta: C

The correct answer is C. In hypochondriasis, the patient has the firm conviction of having a serious illness, despite repeated evidence to the contrary. Even when presented with definitive evidence, these patients remain convinced that they are ill. Oftentimes these patients make multiple visits to different physicians and undergo repeated diagnostic studies, but still remain certain that they have a serious affliction.

In body dysmorphic disorder (choice A), the preoccupation has to do with physical appearance, not a physical illness.

In conversion disorder (choice B), a deficit or symptom of voluntary motor or sensory function suggests a neurologic disease, which is not present.

Schizophrenia (choice D) involves delusions and/or hallucinations, and onset is usually much earlier in life.

Somatization disorder (choice E) is characterized by multiple somatic complaints. In hypochondriasis, there is the fear that one has a particular disease.

How well did you know this?
1
Not at all
2
3
4
5
Perfectly
6
Q

A 68-year-old Latina is seen in the emergency department because of complaints of anxiety. She states she has been taking medication for anxiety for the past 10 years; however, she ran out of medication 2 days ago. She denies the use of any illicit drugs or alcohol. She does not know the name of the medication she has been taking. Laboratory studies are remarkable for a urine drug screen positive for benzodiazepines. The physician should be most concerned about withdrawal from which of the following medications?

(A) Alprazolam
(B) Buspirone
(C) Chlordiazepoxide
(D) Clonazepam
(E) Diazepam

A

Respuesta: A

The correct answer is A. Of the four benzodiazepines listed, alprazolam has the shortest half-life and the greatest potential for precipitating withdrawal. Benzodiazepine withdrawal, which is indistinguishable from alcohol withdrawal, can be life-threatening.

Buspirone (choice B) is not a benzodiazepine and is not associated with a known withdrawal syndrome. It has FDA approval for generalized anxiety disorder.

Chlordiazepoxide (choice C), clonazepam (choice D), and diazepam (choice E) are all benzodiazepines with long half-lives. Although abrupt cessation of these medications may precipitate withdrawal, it would be less likely. Therefore, given the choices listed, one would choose the benzodiazepine with the shortest half-life as being the medication most likely to be of concern to the physician.

How well did you know this?
1
Not at all
2
3
4
5
Perfectly
7
Q

A mother brings her 4-year-old boy to the pediatrician because he has had trouble relating to children in his new preschool. His birth history was unremarkable, but he has been slow to develop language and has required speech therapy. In the office, the child rarely makes eye contact, and he flaps his hands. When given a doll, he does not engage in imaginary play. He has no dysmorphic features, and the rest of his examination is normal. Which of the following is the most likely diagnosis?

(A) Asperger syndrome
(B) Autism
(C) Dyslexia
(D) Fragile X syndrome
(E) Rett syndrome

A

Respuesta: B

The correct answer is B. The diagnosis of autism requires impairment in social interaction and language combined with repetitive or stereotyped patterns of behavior such as hand flapping. There must also be a history of delayed development, occurring prior to age 3 years, in one of the following domains: social interaction, communication, or imaginative play. The etiology is currently unknown, but studies of monozygotic twins suggest a strong genetic component. The most common genetic cause of autism is the fragile X syndrome.

Asperger syndrome (choice A) is similar to autism but spares language function. A patient must have normal language development in the presence of both repetitive or stereotyped behaviors and impaired social interaction.

Dyslexia (choice C) is a language disorder predominantly affecting reading. It may be heralded by a history of language delay in children who are not yet old enough to read, as in this case, but there should be no repetitive behaviors or impaired social interaction.

Fragile X syndrome (choice D) is a common X-linked recessive disorder involving a trinucleotide repeat. It affects boys much more severely than girls, who are usually normal or minimally impaired. It is a common genetic cause of both mental retardation and autism. In addition to symptoms of these two conditions, boys with fragile X often have dysmorphic features, most commonly a long face and enlarged ears.

Rett syndrome (choice E) shares many features of autism, but it affects only girls. In addition to the autistic features, girls with this disorder develop deceleration of head growth between 5 and 48 months of age, lose previously acquired hand skills, and show poor coordination of gait or trunk movements on exam.

How well did you know this?
1
Not at all
2
3
4
5
Perfectly
8
Q

A 43-year-old African American woman is discharged from a 4- week-long alcohol dependence treatment program. At the end of this program, the patient discusses with her physician possible pharmacologic modalities that may assist in preventing relapse. It is decided that she begin a trial of disulfiram (Antabuse™). She is told that she must avoid ingestion or dermal contact of any substances containing alcohol. The patient asks how the medication works and is told that disulfiram inhibits an enzyme that then causes accumulation of a noxious metabolite. Inhibition of which of the following enzymes accounts for the clinically significant effect of disulfiram?

(A) Acetylcholinesterase
(B) Alcohol dehydrogenase
(C) Aldehyde dehydrogenase
(D) Cytochrome P450
(E) Tyrosine hydroxylase

A

Respuesta: C

The correct answer is C. Disulfiram inhibits aldehyde dehydrogenase, which causes a marked increase in levels of acetaldehyde after consumption of alcohol, which in turn causes a wide range of unpleasant effects, such as vomiting, headache, and dyspnea. Its success is predicated on the degree of motivation on the part of the patient to abstain from alcohol, as its efficacy is entirely dependent on compliance.

Acetylcholinesterase (choice A) inhibitors used in psychiatry include donepezil and tacrine. These are used to treat Alzheimer type dementia.

Alcohol dehydrogenase (choice B) catalyzes the metabolism of ethanol to acetaldehyde. Disulfiram does not inhibit this first step in alcohol metabolism.

Several medications affect the cytochrome P450 (choice D) system, necessitating appropriate adjustments in dosing. Disulfiram’s mechanism of action, however, is at the site of the aldehyde dehydrogenase enzyme.

Tyrosine hydroxylase (choice E) converts tyrosine into DOPA, a neurochemical conversion unrelated to the mechanism of action of disulfiram

How well did you know this?
1
Not at all
2
3
4
5
Perfectly
9
Q

On Halloween, a group of teenagers decides to go to a new haunted house that they heard had extraordinary “scary” effects. At the entrance, they laugh loudly and tease each other. This is an example of which of the following types of behavior?

(A) Counterphobic behavior
(B) Denial
(C) Reaction formation
(D) Regression
(E) Undoing

A

Respuesta: A

The correct answer is A. Counterphobic behavior refers to seeking out situations or objects that are or were feared. The person actually takes a position of actively attempting to confront and master what he or she fears.

Denial (choice B) is the avoidance of awareness of a painful aspect of reality by negating sensory data and thus abolishing external reality. Denial may be seen in both normal and pathologic states.

Reaction formation (choice C) transforms an unacceptable impulse into its opposite. It is used in all stages of development and is typically seen in obsessive disorders.

Regression (choice D) is a type of behavior through which a person attempts to avoid tension and conflict at a present level of development by going to an earlier libidinal phase of functioning. It can be normal or can reflect the need to gain easier gratification at a less developed stage.

Undoing (choice E) is a secondary defensive operation that a person uses to undo or prevent the consequences that are anticipated irrationally as a result of unacceptable thought or impulse. It is mostly seen in obsessive disorders.

How well did you know this?
1
Not at all
2
3
4
5
Perfectly
10
Q

A 40-year-old man presents to the emergency department complaining of abdominal pain. He lives alone and states that he was otherwise healthy before this episode. He undergoes emergency abdominal surgery for the removal of a ruptured appendix. Three days after surgery, the patient becomes delirious, with fluctuations in his level of consciousness, but is afebrile. Which of the following would be the most likely source of this patient’s delirium?

(A) Delirium tremens
(B) Infection
(C) Pain medications
(D) Postoperative depression
(E) Stress of surgery

A

Respuesta: A

The correct answer is A. Delirium tremens is the most common cause of delirium in a patient who is suddenly admitted to the hospital for an unrelated condition and no longer has access to alcohol. It is a medical emergency, with a 20% mortality rate if left untreated. It usually occurs within 1 week after the patient stops drinking.

Infection (choice B) is a complication leading to delirium after surgery, but it is commonly associated with a high fever.

Pain medications (choice C) can cause delirium, but usually in elderly patients whose metabolic functions are compromised.

Postoperative depression (choice D) is not a known cause of delirium.

The stress of surgery (choice E) is a cause of postoperative delirium, but it is less common and usually occurs in procedures such as organ transplants.

How well did you know this?
1
Not at all
2
3
4
5
Perfectly
11
Q

A 40-year-old Caucasian man with chronic paranoid schizophrenia presents to his primary care physician’s office for his yearly physical examination. The physician notes that the patient currently weighs 220 pounds, whereas the previous year he weighed 197 pounds. The patient explains to the physician that his psychiatrist switched his medication about 5 months earlier, from risperidone to a new medication called olanzapine. The patient states that over the past few months, he has had a decrease in auditory hallucinations but has had some difficulty because he was laid off from his job and has been having increasing strain in his marriage. He states also that he has been drinking more alcohol lately, up to a case of beer a week. Which of the following is the most likely cause of this patient’s increase in weight?

(A) Alcohol use
(B) Decreased auditory hallucinations leading to increased appetite
(C) Discontinuation of risperidone
(D) Initiation of olanzapine
(E) Overeating due to familial stress

A

Respuesta: D

The correct answer is D. Olanzapine, an atypical antipsychotic, has been demonstrated to lead to weight gain in many patients with schizophrenia, with studies showing a gain of up to 27 pounds in the course of a year. Olanzapine affects the 5HT2 serotonin receptor in the brain, which is also thought to control satiety, in addition to decreasing auditory hallucinations and controlling mood symptoms.

Alcohol use (choice A) can lead to weight gain but would not have caused the significant gain in this patient, as he only recently began drinking more.

Decreased auditory hallucinations (choice B) have not been shown to have a significant correlation to appetite and subsequent weight gain.

There is no increase in appetite or weight associated with the discontinuation of risperidone (choice C).

Although some patients do overeat in response to stress, this activity is more common in female than in male patients. In addition, this patient describes his familial problems (choice E) as more recent in their onset; therefore, this would not be the most likely cause for his weight gain.

How well did you know this?
1
Not at all
2
3
4
5
Perfectly
12
Q

A 4th-year medical student is interviewing a 38-year-old woman who has schizophrenia, disorganized type. As part of the mental status examination, the student asks the patient to name the floor of the hospital she is on. In response, the patient begins to discuss the last hospital she was admitted to, the quality of the food while she was a patient there, and how attractive she thought her last physician was. Her speech is intelligible and normal in tone, and her vocabulary is good, but she never answers the student’s question. Which of the following best describes this patient’s behavior?

(A) Circumstantiality
(B) Tangentiality
(C) Thought blocking
(D) Verbigeration
(E) Word salad

A

Respuesta: B

The correct answer is B. Tangentiality is a disturbance in communication characterized by a lack of goal-directed association of thoughts. Patients cannot mentally get from a desired starting point to a desired goal (in this case an answer as to which floor of the hospital the patient is on), but instead jump from one topic to another.

Circumstantiality (choice A) is speech in which a patient eventually reaches the answer to a question, but frequently digresses along the way before arriving back at the central idea.

Thought blocking (choice C) is an abrupt interruption in a patient’s logical progression of thought before an idea or thought is finished.

Verbigeration (choice D) is the meaningless repetition of specific words or phrases.

Word salad (choice E) is a completely incoherent mixture of words and phrases that have no grammatical meaning in language.

How well did you know this?
1
Not at all
2
3
4
5
Perfectly
13
Q

A 35-year-old woman is admitted to the psychiatric unit for treatment. After a few days on medication, she complains of diarrhea, a metallic taste in her mouth, and polyuria. While examining her, the psychiatrist observes a fine intention tremor of her hands. The patients also complains that her psoriasis has flared up. Which of the following medications most likely caused these symptoms?

(A) Carbamazepine
(B) Fluoxetine
(C) Haloperidol
(D) Lithium carbonate
(E) Valproic acid

A

Respuesta: D

The correct answer is D. This patient is experiencing side effects associated with therapeutic or lower levels of lithium carbonate. These side effects are often troublesome and include sedation, cognitive difficulties, dry mouth, hand tremor, increased appetite, polydipsia and polyuria, nausea, diarrhea, psoriasis, and acne.

The most common side effects associated with carbamazepine (choice A) include dizziness, ataxia, sedation, dysarthria, nausea, hyponatremia, and cardiovascular conduction problems. Its most serious side effects are aplastic anemia and agranulocytosis. Rarely, it can cause rash and exfoliation.

Fluoxetine (choice B) causes nausea, dyspepsia, tremor, nervousness, and increased anxiety during the initial phase of treatment. It has no effect on fluid intake or worsening of dermatologic diseases.

Side effects associated with haloperidol (choice C) typically are a consequence of the blockade of dopaminergic D2 receptors. Apart from a spectrum of extrapyramidal symptoms, side effects include sedation, weight gain, hyperprolactinemia, and possible neuroleptic malignant syndrome.

Valproic acid (choice E) most commonly causes short-term nausea, vomiting, diarrhea, sedation, dizziness, and tremor. Alopecia and weight gain are long-term side effects.

How well did you know this?
1
Not at all
2
3
4
5
Perfectly
14
Q

A 47-year-old Caucasian man is seen by a psychiatrist for treatment of refractory depression. He has a 10-year history of severe depression, which has been treated with several different medications, as well as with electroconvulsive therapy. After careful consideration, the psychiatrist and the patient decide that he should try a monoamine oxidase inhibitor (MAOI). The psychiatrist gives him a list of foods and medications to avoid while taking the MAOI. Which of the following adverse outcomes is the psychiatrist trying to avoid?

(A) Agranulocytosis
(B) Hypertensive crisis
(C) Pigmentary retinopathy
(D) Priapism
(E) Serotonin syndrome

A

Respuesta: B

The correct answer is B. The monoamine oxidase inhibitors (MAOIs) used to treat depression inhibit the A form of the enzyme, which results in increased concentrations of norepinephrine, serotonin, and dopamine wherever the enzyme is present. Dietary tyramine, which is present in foods such as aged cheeses as well as in red wine, can enter the circulation unmetabolized (as alimentary MAO-A is also inhibited) and act as a pressor. If a patient on an MAOI eats foods rich in tyramine or takes certain medications bearing similarities to biogenic amines, a lifethreatening hypertensive crisis could develop.

Agranulocytosis (choice A) is an idiosyncratic adverse event associated with clozapine, among other drugs.

Pigmentary retinopathy (choice C) has been reported in patients taking high (>800 mg/day) doses of thioridazine.

Priapism (choice D) is associated with trazodone use.

Serotonin syndrome (choice E) can occur in individuals taking a serotonin-altering medication (such as an SSRI) in addition to an MAOI. This potentially life-threatening syndrome is characterized by autonomic instability and motor and behavioral abnormalities.

How well did you know this?
1
Not at all
2
3
4
5
Perfectly
15
Q

A 16-year-old girl is brought to the psychiatrist by her mother, who had noticed that her daughter pulls her hair and chews it. It is more evident now that she has several bald patches on both sides of her head. The daughter says that pulling her hair provides some sense of relief when she feels nervous or upset. She is doing well at school and denies any other symptoms. She has seen a dermatologist, who ruled out any medical causes for the alopecia. Which of the following is the most likely diagnosis?

(A) Bulimia nervosa
(B) Factitious disorder
(C) Major depressive disorder
(D) Obsessive-compulsive disorder
(E) Trichotillomania

A

Respuesta: E

The correct answer is E. Trichotillomania is an impulse control disorder. It is usually seen in childhood or adolescence and is most common in girls. It is manifested by repetitive pulling of one’s hair from any part of the body. The tension before pulling is usually relieved afterward. The disturbance is related to impulsive urges, making it different from goal-directed obsessional ideas. The disorder is not due to a general medical condition or the effects of any substance.

Bulimia nervosa (choice A) refers to episodes of binge eating characterized by a sense of lack of control. During those episodes patients take in an amount of food larger than most people would eat. It occurs at least twice a month for 3 months and includes inappropriate behavior to prevent weight gain, such as misuse of laxatives or self-induced vomiting. Bulimia may be comorbid with trichotillomania but does not include hair eating itself.

Factitious disorder (choice B) is intentional production of or feigning of symptoms of illness to assume the sick role. External motivation, like economic gain or avoidance of legal responsibility, is absent.

Major depressive disorder (choice C) requires symptoms of anhedonia and/or depressed mood every day for at least 2 weeks, along with associated symptoms of depression such as low energy, insomnia, weight loss, poor appetite, and poor concentration. It can be comorbid with trichotillomania.

Obsessive-compulsive disorder (choice D) is defined by recurrent intrusive thoughts or impulses that cause marked distress. The person tries to ignore or suppress them or to neutralize them with some action. The patient is aware that those ideas are the product of his or her mind and are unreasonable.

How well did you know this?
1
Not at all
2
3
4
5
Perfectly
16
Q

A 30-year-old Caucasian man has been treated with antipsychotic medications for the past 6 years. He has a chronic delusion that the FBI is constantly monitoring him through hidden cameras in his apartment and listening devices in his telephone. He also claims to hear the voice of the devil telling him not to eat certain foods because his neighbors are trying to poison him. Despite several hospitalizations and multiple trials of different antipsychotic medications, his symptoms are continuously present and are very distressing. Accordingly, his psychiatrist decides to start him on clozapine. Which of the following laboratory studies should be monitored at regular intervals?

(A) Complete blood count
(B) Prolactin level
(C) Serum glucose level
(D) Serum urea nitrogen and creatinine levels
(E) Thyroid stimulating hormone

A

Respuesta: A

The correct answer is A. There is a 1 to 2% risk of agranulocytosis in patients treated with clozapine. This idiosyncratic event can occur at any time in the course of treatment but is most likely to occur within the first 6 months. Subsequently, periodic examination of a white blood cell count with differential is essential while treating with clozapine.

Clozapine has no effect on prolactin levels (choice B) or serum glucose levels (choice C).

There is the possibility of renal toxicity (choice D) and hypothyroidism (choice E) with lithium treatment, not clozapine.

How well did you know this?
1
Not at all
2
3
4
5
Perfectly
17
Q

A 22-year-old African American woman visits a psychiatrist for “violent mood swings.” She reports that she experiences intense periods of “utter contentment” followed, often within minutes to hours, by intense feelings of depression. During these episodes, she feels she is useless and unloved by her parents and boyfriend. In addition, despite desperate attempts to engage in a long-term romantic relationship, she regrets that she can never seem to stay involved with anyone for very long, as she feels “no one understands me.” In assessing past thoughts of harm to herself, she admits that she has had more than 10 suicide attempts, usually involving superficial cuts to her wrists. The patient offers that “sometimes I feel so numb, I cut myself just to feel something.” Which of the following is the most likely diagnosis?

(A) Attention deficit/hyperactivity disorder
(B) Bipolar disorder
(C) Borderline personality disorder
(D) Intermittent explosive disorder
(E) Major depression

A

Respuesta: C

The correct answer is C. One of the hallmark characteristics of borderline personality disorder is affective instability, recognized as rapidly shifting mood states. Other symptoms include unstable and intense interpersonal relationships, chronic feelings of emptiness, and recurrent suicidal thoughts and gestures, often dramatic and attention-seeking in character.

Attention deficit/hyperactivity disorder (ADHD) (choice A) is associated with impulsivity, as is intermittent explosive disorder (choice D); however, mood swings and recurrent suicidality are not essential features.

Bipolar disorder (choice B) is also associated with extremes of mood; however, mood states usually last for days to weeks.

Major depression (choice E) is often comorbid in patients with borderline personality disorder; however, there is insufficient evidence to make such a diagnosis here.

How well did you know this?
1
Not at all
2
3
4
5
Perfectly
18
Q

A 38-year-old African American man with schizophrenia, chronic undifferentiated type, is evaluated by a psychiatrist. On mental status examination, he has a notable poverty of speech, along with poor eye contact, inattentiveness during testing, thought insertion, and flat affect. He does not appear to be actively responding to auditory hallucinations. Which of the above is the only positive symptom of schizophrenia in this patient?

(A) Flat affect
(B) Inattentiveness during testing
(C) Poor eye contact
(D) Poverty of speech
(E) Thought insertion

A

Respuesta: E

The correct answer is E. There is a clinical distinction in patients with schizophrenia between positive (or productive) symptoms of schizophrenia and negative (or deficit) symptoms. Although not accepted as part of the DSM-IV classification, the clinical distinction of the two types has significantly influenced psychiatric research. Positive symptoms of schizophrenia include delusions, hallucinations, thought insertion, and thought broadcasting.

Negative symptoms include affective flattening (choice A), deficits in attention (choice B), poor eye contact (choice C), and alogia, which is the lack of ability to produce normal fluent speech (choice D).

How well did you know this?
1
Not at all
2
3
4
5
Perfectly
19
Q

A 27-year-old man is brought to the emergency department in an agitated state by police. The man had been talking to himself and threatening people in the street, stating he was the son of God. On the psychiatric unit, he refuses to take medication. However, after engaging in several altercations with other patients, he is given an injection of haloperidol. The haloperidol is then ordered “as needed” in case of agitation over the next few days. The staff reports that he has started pacing down the hall and cannot sit still for more than 5 minutes. Which of the following disorders is this patient most likely experiencing?

(A) Akathisia
(B) Akinesia
(C) Festination
(D) Stereotypic movement disorder
(E) Tardive dyskinesia

A

Respuesta: A

The correct answer is A. Akathisia denotes a state of extreme motor restlessness. The patient cannot sit still and is constantly walking, shifting weight, and pacing. It is seen in encephalitic illnesses and as a complication of neuroleptic treatment.

Akinesia (choice B) is a term that refers to a failure of the patient to engage the limbs in customary activities. It results in poverty of movement, including small automatic postural adjustments, as well as in volitional movements. It is seen in Parkinson disease and related disorders.

Festination (choice C) is a gait disorder characteristic of Parkinson disease. The patient’s trunk is bent forward, the arms are slightly flexed, the legs are bent at knees, and steps are short and shuffling. With walking, the upper body advances ahead of the lower and steps become increasingly rapid to “catch up.”

Stereotypic movement disorder (choice D) is repetitive nonfunctional motor behavior (rocking, head banging, self-biting, picking) lasting at least 4 weeks. It is usually seen in mental retardation or pervasive developmental disorder, and it significantly interferes with normal activities.

Tardive dyskinesia (choice E) is a late-appearing disorder of involuntary, choreoathetoid movements of the orofacial region or trunk and limbs, following neuroleptic treatment. The movements are present at least 4 weeks. The neuroleptic treatment must last at least 3 months, or the movements must develop within 4 weeks after withdrawal from treatment

How well did you know this?
1
Not at all
2
3
4
5
Perfectly
20
Q

The 12th grade teacher of a 17-year-old girl is concerned about the girl’s behavior over the past school year. She has been talking to herself while looking off into the distance, isolating herself from her classmates, and wearing the same clothes daily for several weeks likely without washing them. However, she has been completing her homework assignments. The teacher calls the girl’s parents, who admit that she has been “acting strangely” for quite a while. The parents ask their daughter if anything is bothering her. She replies “I want to save Swedish children from persecution by the devil.” According to data from the National Institute of Mental Health Sponsored Epidemiologic Catchment Area, what is the lifetime prevalence of this girl’s disorder?

(A) 0.01%
(B) 0.1%
(C) 1%
(D) 5%
(E) 10%

A

Respuesta: C

The correct answer is C. This girl has schizophrenia, which has a lifetime prevalence of 0.6 to 1.9% (1% is most often cited as the average lifetime prevalence). About 0.025 to 0.05% of the total population is treated for schizophrenia in any single year. It is equally prevalent in men and women; however, the onset and course of illness differ between the sexes. Onset is earlier in men than in women. The peak age for men is 15-25 years, and the peak age for women is 25-35 years. In general, the outcome for patients with schizophrenia is better for women than for men; some studies have suggested that men are more likely to be impaired with negative symptoms and have poorer social functioning. Interestingly, the use of psychotherapeutic drugs, the deinstitutionalization of state hospitals, the emphasis on rehabilitation, and the community-based care for schizophrenic patients have led to an increase in the marriage and fertility rates among people with schizophrenia. Because of this, the number of children with at least one schizophrenic parent recently doubled.

The lifetime prevalence for major depressive disorder is 10 to 25% for women and 5 to 12% for men. The lifetime prevalence for dysthymic disorder is 6%. The lifetime prevalence for bipolar I disorder is 0.4 to 1.6%; for bipolar II disorder, it is 0.5%. The lifetime prevalence for cyclothymic disorder is 0.4 to 1.0%.

How well did you know this?
1
Not at all
2
3
4
5
Perfectly
21
Q

A 28-year-old pregnant woman attempts to drown herself and her three young children. She is brought to the emergency department, and her children are taken to their grandmother’s house. The woman is offered admission but she refuses, saying, “It doesn’t matter. It’s all going to end sooner or later.” Which of the following is the most appropriate next step in management?

(A) Admit her to a day program and order 15-minute observation checks there
(B) Admit her as an involuntary patient to an inpatient unit with high observation
(C) Arrange for outpatient follow-up to begin the following morning
(D) Create a contract with her to ensure that she agrees not to harm herself
(E) Call the police to report her as having endangered her children and unborn fetus

A

Respuesta: B

The correct answer is B. For a patient to be admitted involuntarily to an inpatient psychiatric unit, she must show evidence of either being harmful to others or to herself (either by a suicide attempt, self-mutilation, or selfneglect). This patient has clearly acted as a danger not only to herself but also to others. She may be suffering from depression, manic-depression, psychosis, substance abuse, obsessive-compulsive disorder, posttraumatic stress disorder, or other psychiatric conditions. She needs a thorough evaluation on a highly monitored inpatient psychiatry unit to better understand her condition.

Choices A and C are unacceptable because it is too dangerous for this patient to leave the hospital. Ultimately she may benefit from a day program and/or from outpatient care but certainly not at this point. Also, if the clinician believes that a patient is dangerous enough to require observation checks at the frequency of every 15 minutes, the patient most likely needs around-the-clock inpatient care.

Studies have shown that written contracts with psychiatric patients (choice D) are usually worthless. Their only consistently demonstrated value is for the evaluation of a patient who states that she is unable to contract for safety. Under these circumstances, it is made clear to the staff that the person does not trust herself to stay safe.

Calling the police to report her as having endangered her children and unborn fetus (choice E) may be necessary at some point, but the priority in the emergency department is to ensure the patient’s safety, and this must be through an involuntary hospitalization

22
Q

A 40-year-old woman presents to the emergency department after having cut her arm on a knife. She states that she sleeps with a knife for protection from her neighbors, who she thinks are conspiring to rob her. The patient also tells the physician that she cannot hold a job because everywhere she works, her co-workers conspire to slander her for her political beliefs. Which of the following is the most likely diagnosis?

(A) Capgras syndrome
(B) Delirium
(C) Delusional disorder, persecutory type
(D) Paranoid schizophrenia
(E) Shared psychotic disorder

A

Resultado: C

The correct answer is C. The above symptoms are suggestive that this woman has delusional disorder, persecutory type, as her beliefs are not outside the realm of reason but are probably not based in reality. Her feelings that other people are out to hurt or injure her put the delusional disorder in the realm of persecutory type.

Capgras syndrome (choice A) is the delusion that others, or oneself, have been replaced by impostors. It typically follows the development of negative feelings toward the other person or self that the subject cannot accept and instead attributes to an impostor. The syndrome is frequently found in organic brain disease.

Delirium (choice B) requires an alteration of the state of consciousness, which this patient does not have.

Paranoid schizophrenia (choice D) is a diagnosis that requires that the patient have some of the symptoms of schizophrenia, including auditory hallucinations, thought blocking, or blunted affect. The patient’s age and her ability to obtain employment make schizophrenia an unlikely diagnosis in this case.

In shared psychotic disorder (choice E), the patient has a system of disordered thought that is assumed by another person. In most cases, this disorder is found among spouses; typically the spouse with the psychotic disorder is more dominant in the social relationship.

23
Q

A 24-year-old physics student with a history of bipolar disorder presents to a clinic. He is dressed in new white suit and is wearing a hat. He goes around the waiting room, flirting with female patients, laughing, and talking loudly. Which of the following delusions is most consistent with his condition?

(A) “I created the bomb that killed people in Vietnam; I don’t deserve to live.”
(B) “I know that my family is going bankrupt; we won’t have anything to eat.”
(C) “I heard them talking about me on TV; they clearly said my name.”
(D) “My brain is melting inside; I know it.”
(E) “NASA called me to be the director of their new space program.”

A

Respuesta: E

The correct answer is E. The patient is presenting with a fullblown mania, in which delusions of grandeur are common and congruent with euphoric elevated mood.

Delusions of self-accusation (choice A) are false feelings of remorse and guilt frequently seen in depressive disorder with psychotic features.

Delusions of poverty (choice B) are false beliefs that one is bereft or will be deprived of all the material possessions.

Delusions of reference (choice C) are false beliefs that one is being talked about by others and that events or people have unusual significance in relationship to oneself.

Somatic delusion (choice D) is a false belief involving the function of one’s body and can be seen in psychotic disorders of schizophrenic spectrum, as well as in depressive disorder with psychotic features.

24
Q

A 31-year-old man describes himself as “on top of the world” to strangers whom he meets in the street or on public transportation. He is considering running for president of the United States, since he views himself as more intelligent, handsome, and charismatic than anyone he knows. He has engaged in sexual activity with eight different women over the past 8 nights. He thinks his talents are unique and are the envy of all. He has been prescribed lithium by his treating psychiatrist, yet he has opted not to take the medication because he feels that it limits his creativity and genius. Which of the following symptoms would be seen in such a person during a manic episode but not during a major depressive episode?

(A) Anhedonia
(B) Decreased sleep
(C) Distractibility
(D) Flight of ideas
(E) Impairment of functioning

A

Respuesta: D

The correct answer is D. This patient has bipolar disorder. He is currently in a manic episode, characterized by at least a week (at least 8 days in his case) of inflated self-esteem, pressure to talk, and a goal-directed focus on a grandiose wish to be president. He also has engaged in excessive involvement in sexual indiscretions without regard for the potential dangerous outcomes. Further, although not described in the question, the patient likely has a decreased need for sleep, likely has a subjective feeling that his thoughts are racing, and likely has a wavering attention span. Flight of ideas is characteristic of mania but not of depression. A flight of ideas is a rapid jumping of thoughts through a series of tenuously related ideas that make more sense to the manic patient than to the examiner.

Anhedonia (choice A) is characteristic of depression but not mania. Anhedonia is characterized by the absence of pleasure from other persons, situations, or things that would ordinarily be pleasurable. During a manic episode, persons often experience the opposite of anhedonia by finding pleasure in things that do not typically evoke such a contented response.

Decreased sleep (choice B) is usually characteristic of both mania and depression. In mania, persons often do not feel as though they need a lot of sleep. In depression, persons have difficulty in both falling asleep and staying asleep. Early morning awakening with an inability to fall back asleep is very common in depression. The net result for both mania and depression typically is decreased sleep.

Distractibility (choice C) can be an attribute of both mania and depression. In manic individuals, their attention is often too easily drawn to unimportant or irrelevant external stimuli. In depressed individuals, they are usually unable to focus on completing any one task, since all tasks are often perceived as too difficult to complete. These people are often distracted by their own sad moods.

Impairment of functioning (choice E) is among the DSM-IV list of criteria for both a manic episode and a major depressive episode.

25
A 52-year-old woman with a history of mitral valve prolapse, myocardial infarction, and diabetes returns to her primary care physician’s office after having been recently diagnosed by her psychiatrist as having panic disorder. She is obese, has a history of alcohol dependence, and frequently has episodes of hypoglycemia due to poor control of blood glucose. Which of the following conditions has a demonstrated association with panic disorder? (A) Alcohol dependence (B) Diabetes (C) Hypoglycemia (D) Mitral valve prolapse (E) Obesity
Respuesta: A The correct **answer is A**. Panic disorder is characterized by the spontaneous, unexpected occurrence of panic attacks. Panic disorder is often accompanied by agoraphobia, the fear of being alone in public places. Alcohol and other substance dependence occurs in about 20 to 40% of all patients with panic disorder. Diabetes (**choice B**) and hypoglycemia (**choice C**) are incorrect because neither low nor high blood sugar levels are known to cause psychiatric disorders, although many patients believe that panic disorder is caused by hypoglycemia. Mitral valve prolapse (**choice D**) is incorrect because, although many people believe that mitral valve prolapse leads to panic disorder, there is no evidence that panic disorder is more prevalent in patients with mitral valve prolapse than in the general population. There is no established correlation between the incidence of obesity (**choice E**) and the incidence of panic disorder in the general population.
26
An 87-year-old Hispanic man is accompanied by his daughter for evaluation of memory problems. The daughter explains that, for the past year, her father has been getting increasingly forgetful. He misplaces personal items, such as his house keys, and it often takes him a while to come up with the words for simple objects. Furthermore, he has been acting suspicious toward his wife, whom he suspects is having an affair even though she is bedridden. Last week, he insisted that his apartment had been broken into because he discovered that the entry door was unlocked one morning when he awoke. When his wife suggested that he might have forgotten to lock it, he became very angry and said, “it was probably that man you’ve been going around with!” Physical examination is unremarkable, and laboratory studies, including a thyroid stimulating hormone (TSH) level, are within normal limits. Which of the following is the most likely underlying pathology that accounts for this patient’s symptoms? (A) Bilateral necrosis of the globus pallidus (B) Focal demyelination and gliosis of periventricular white matter (C) Focal hemorrhage and necrosis of the mammillary bodies (D) Loss of pigmented cells in the substantia nigra (E) Neurofibrillary tangles and senile plaques in the temporoparietal cortex
Respuesta: E The correct **answer is E**. The clinical vignette is illustrative of dementia of the Alzheimer type. Not only are there typical signs of memory loss, but there may also be evidence of paranoid ideation or delusions, especially as the disease advances. Microscopic examination of postmortem brain tissue reveals the presence of neurofibrillary tangles and senile plaques, which are present to a lesser extent in the brains of nondemented elderly individuals. It is the correlation between the patient’s presentation and the abundance of these microscopic features that ensures the diagnosis. Necrosis of the globus pallidus (**choice A**) is suggestive of carbon monoxide poisoning, which is acute in presentation and may result in an amnestic syndrome with affective disturbance. Laboratory test shows elevated carboxyhemoglobin levels. Focal demyelination and gliosis of periventricular white matter (**choice B**) are consistent with multiple sclerosis. Initially, symptoms of paresthesias, gait disturbance, motor weakness, and visual effects generally begin before the age of 55, and women are slightly more often affected than are men. Involvement of the mammillary bodies is consistent with Wernicke encephalopathy (**choice C**), which results from severe thiamine deficiency and is most often associated with chronic alcoholism. Wernicke encephalopathy is characterized by the triad of confusion, ataxia, and nystagmus leading to ophthalmoplegia. Parkinson disease is microscopically characterized by loss of pigmented cells in the substantia nigra (**choice D**). Clinically, it is characterized by tremor, rigidity, hypokinesia, and abnormal gait and posture.
27
A 32-year-old archeologist has been fighting with his wife because of his drug abuse. His wife is threatening to divorce him and take their child unless he quits using drugs. To keep his family together, he decides to stop using drugs right away, without going to treatment. Soon afterward, he has cramps and diarrhea, is sneezing and yawning, and stays wrapped in the blanket feeling chills and aches all over his body. Which of the following drugs was this man most likely taking? (A) Alprazolam (B) Amphetamines (C) Cocaine (D) Heroin (E) Nicotine
Respuesta: D The correct **answer is D**. Opioid withdrawal follows cessation or reduction of prolonged and heavy opioid use and includes at least three of the following symptoms: dysphoric mood, nausea or vomiting, muscle aches, yawning, diarrhea, lacrimation, rhinorrhea, piloerection, pupillary dilatation, sweating, fever, and insomnia. The symptoms cause significant distress in social or occupational functioning and are not due to a general medical condition. Alprazolam (**choice A**) is a benzodiazepine. Withdrawal starts after a reduction or cessation of prolonged alprazolam use. Symptoms may include autonomic hyperactivity, increased hand tremor, insomnia, nausea and vomiting, transient hallucinations, psychomotor agitation, grand mal seizures, or anxiety. Amphetamine (**choice B**) withdrawal follows a cessation of prolonged and heavy amphetamine use. The symptoms include dysphoric mood and physiologic changes, such as fatigue, unpleasant dreams, insomnia or hypersomnia, increased appetite, psychomotor retardation, or agitation. Cocaine (**choice C**) withdrawal follows a cessation of heavy and prolonged cocaine use. The symptoms include dysphoric mood and physiologic changes, such as fatigue, increased appetite, psychomotor retardation or agitation, insomnia or hypersomnia, or vivid unpleasant dreams. Nicotine (**choice E**) withdrawal starts within 24 hours after abrupt cessation of nicotine use. Withdrawal signs include dysphoric mood, insomnia, irritability, difficulty concentrating, anxiety, restlessness, decreased heart rate, and increased appetite.
28
A 32-year-old man is diagnosed with obsessive-compulsive disorder by his psychiatrist, and is started on daily sertraline. Which of the following is the most common compulsion in patients with this disorder? (A) Avoiding social situations (B) Incessant checking (C) Overwhelming need to repeat work tasks (D) Repetitive washing (E) Striving for completeness
Respuesta: D The correct **answer is D**. The most common compulsion associated with obsessive-compulsive disorder (OCD) is excessive or ritualized hand washing, showering, bathing, toothbrushing, or grooming, with a reported rate of nearly 60% of all patients. The typical pharmacologic treatment for OCD is a selective serotonin reuptake inhibitor (SSRI) such as sertraline. Avoidance of social situations (**choice A**) is not a compulsion. It is more a symptom of avoidant personality disorder than of OCD. Incessant checking (**choice B**) occurs in about 32% of all OCD patients. Repeating rituals (**choice C**) occurs in about 36% of all OCD patients. Striving for completeness (**choice E**) is a nonpathologic trait that is not indicative of psychiatric illness in the general population
29
A 76-year-old Caucasian male, accompanied by his wife, presents at a psychiatrist’s office. The man has a 5-year history of depression and has tried several antidepressant medications without satisfaction. At the advice of his primary care physician, the patient was referred to the psychiatrist to be evaluated for electroconvulsive therapy (ECT). After a thorough discussion regarding the treatment, including its efficacy, risks, and benefits, the patient’s wife asks whether there are any absolute contraindications to ECT. Which of the following is the most appropriate response? (A) “Anyone who has ever had a heart attack should not receive ECT.” (B) “Only pregnant patients should not receive ECT.” (C) “Patients with a seizure disorder are not candidates for ECT.” (D) “There are no absolute contraindications to receiving ECT.” (E) “Very elderly patients should not receive ECT.”
Respuesta: D The correct **answer is D**. It is generally accepted that there are no absolute contraindications to receiving electroconvulsive therapy (ECT). There are situations in which risk is increased and close monitoring is prudent; however, any patient is a potential candidate for ECT. Although patients who have had a recent myocardial infarction (**choice A**) are at higher risk for complications from ECT, this risk is significantly reduced 3 months after the event. Therefore, it would be inappropriate to say that any patient who has ever had a myocardial infarction should never receive ECT. Pregnancy (**choice B**) is incorrect; in fact, ECT may be the most appropriate treatment for a variety of disorders in a first-trimester pregnant patient, for whom certain medications may pose a teratogenic risk to the fetus. Although, ideally, a patient should discontinue anticonvulsants during ECT, patients with a seizure disorder on anticonvulsants can safely be treated with ECT (**choice C**), often with excellent results. In fact, one indication for ECT is for intractable seizures. The thought is that repeated seizures actually increases seizure threshold. Age itself (**choice E**) is not an absolute or relative contraindication to ECT. ECT may, in fact, be the only reasonable option in an elderly patient who may otherwise not tolerate psychotropic medications. ECT is frequently used and is very safe in geriatric populations.
30
A young woman has been in psychotherapy twice a week for several months. She feels “used” by others and gets frustrated when she is unable to refuse to do favors for people, even though she knows that they do not deserve her help. On one occasion, her psychotherapist is very late for the session because of an urgent situation. At the next session, the patient brings him a small gift. Which of the following mechanisms is most likely present in this situation? (A) Blocking (B) Displacement (C) Inhibition (D) Isolation (E) Reaction formation
Respuesta: E The correct **answer is E**. Reaction formation is a defense in which an unacceptable impulse or feeling becomes transformed into its opposite. It is mostly seen in obsessive behavior but can be seen in other forms of neurosis. Blocking (**choice A**) is a temporary inhibition of thinking. It also applies to affect and impulses. It is similar to repression, except that the patient feels tense once blocking of thoughts, feelings, or impulses is inhibited. Displacement (**choice B**) occurs when an emotion is shifted from one idea or object to another. It permits symbolic representation of the original idea in a way that causes less distress than the original. Inhibition (**choice C**) is a conscious process by which ego evades anxiety arising from conflicts with instinctual impulses, the superego, or external stressors. Isolation (**choice D**) is the separation of an idea from the affect that accompanies it. The affect remains repressed while the patient talks freely about the idea.
31
The family of a 42-year-old violinist brings her to a psychiatrist for evaluation. She tells the psychiatrist that a famous Italian conductor is in love with her and is planning to leave his wife so they can be married. She had met him once at a reception while on tour in Milan with the philharmonic orchestra. She further tells the psychiatrist that, during his concerts, he gives her signals that he loves her. In addition, the family reports that she somehow got his email address and has been sending him messages. Which of the following types of delusional disorder does this patient most likely have? (A) Erotomanic (B) Jealous (C) Grandiose (D) Persecutory (E) Somatic
Respuesta: A The correct **answer is A**. In the erotomanic type of delusional disorder, the central delusion evolves around the patient’s belief that a famous person or a superior is intensely in love with the patient. Erotomanic patients usually harass public figures (objects of their delusion) through letters, calls, gifts, or visits. The disorder is more frequent in women who are single and have had limited intimate contacts and a modest, isolated way of life. The onset is usually after the age of 40, but can be earlier. In jealous type delusional disorder (**choice B**), the central delusion concerns the fidelity of the spouse. Men are more affected, and the disorder can lead to significant abuse, following a search “for evidence.” At times, the accusation of infidelity results in the murder of the “accused” spouse. There is frequent association with alcoholism. Grandiose delusions (**choice C**) are associated with megalomania and mostly involve the belief of having made important discoveries, or having unrecognized talents. At times they may have a religious content, and the patients become leaders of religious cults. Persecutory delusions (**choice D**) are the most common type and involve the single idea of being conspired against, spied on, followed, or obstructed in achievement of long-term goals. As a result of their delusion, patients may become angry and violent. The somatic type delusional disorder (**choice E**) is also known as monosymptomatic hypochondriacal psychosis. Patients are convinced that they have a presumed illness, most commonly infection, or some body odor. The disorder is seen in both sexes and is differentiated from hypochondriasis by the degree of conviction.
32
A 20-year-old male college student is brought to the emergency department by his roommate for increasingly odd behavior. The patient has grown increasingly isolated over the past 6 months, with little interest in socializing. Four weeks ago, he began accusing his roommate of trying to “steal thoughts” from his head. His schoolwork has become increasingly disorganized, and he has missed several deadlines for papers. On the day of admission, the roommate heard the patient talking loudly to himself. His initial examination and a head CT are unremarkable. On psychiatric evaluation, he is noted to have a bizarre affect and reports hearing voices. Which of the following is the most likely diagnosis? (A) Bipolar disorder (B) Delusional disorder (C) Depression with psychotic features (D) Schizophrenia (E) Schizophreniform disorder
Respuesta: D The correct **answer is D**. The diagnosis of schizophrenia requires that at least two of the following symptoms be present for at least a 1-month period: delusions, hallucinations, disorganized speech, disorganized or catatonic behavior, or flattened affect. In addition, a milder degree of these symptoms must be present for at least 6 months. Symptoms must be severe enough to interfere with work or school performance. The peak age of onset for men is 15-25. Imaging is typically normal but may show enlarged lateral ventricles. Bipolar disorder (**choice A**) may have a similar age of onset and may present with psychotic features, but there is no indication of prominent manic or depressive behavior in this case. Patients with delusional disorder (**choice B**) report less bizarre delusions (i.e., “people at work are doctoring my reports to get me fired”) than are typically reported in schizophrenia. The delusions in this disorder occur in isolation, so that other symptoms of schizophrenia (e.g., disorganized behavior, hallucinations) should be absent. Depression with psychotic features (**choice C**) can present with prominent auditory hallucinations and delusions but on a background of major depression. Typically, patients have a substantial history of depression prior to developing psychotic features. Schizophreniform disorder (**choice E**) is diagnosed when a patient meets all criteria for schizophrenia but has had symptoms for only 1-6 months. Most of these patients go on to develop schizophrenia.
33
A 42-year-old teacher at a local school is brought to the emergency department by her husband and brother because she suddenly lost her voice. The day before, she learned about her husband’s affair with his secretary. While the physician is asking questions and talking to her and the family members, she seems unconcerned with this sudden onset of illness. Which of the following terms is typically used to describe her lack of concern? (A) Denial (B) Depersonalization (C) Displacement (D) Dissociation (E) La belle indifference
Respuesta: E The correct **answer is E**. The outward lack of concern for a physical illness is called la belle indifference. The term was introduced by Pierre Janet and is mostly seen in conversion disorder. Denial (**choice A**) is the avoidance of the awareness of a painful aspect of reality by negating sensory data and thus abolishing external reality. Denial may be seen in normal as well as in pathologic states. Depersonalization (**choice B**) is characterized by persistent recurrent episodes of feeling detached from one’s self or body, like feeling mechanical or in a dream. Reality testing remains intact during those episodes. It can happen in depersonalization disorder or as a part of other mental disorders. Displacement (**choice C**) occurs when an emotion is shifted from one idea or object to another. It permits symbolic representation of the original idea in a way that causes less distress than the original. Dissociation (**choice D**) is a way that an individual deals with emotional conflict or stressors with a breakdown in the integrated functions of consciousness, memory, or perception of self or environment.
34
A 43-year-old man with a history of paranoid schizophrenia presents to his psychiatrist for a yearly medication check. He is currently taking haloperidol at bedtime. The psychiatrist notices worsening tardive dyskinesia since last year, with an increase in abnormal involuntary movements in the hands and perioral area. Which of the following would be the most effective treatment in controlling this patient’s tardive dyskinesia? (A) Addition of an anticholinergic drug (B) Addition of a dopamine receptor antagonist (C) Making no changes in the patient’s antipsychotic medication (D) Stopping the patient’s antipsychotic medication (E) Substituting a serotonin-dopamine antagonist for the haloperidol
Respuesta: E The correct **answer is E**. Tardive dyskinesia is a syndrome of abnormal involuntary movements often associated with long-term neuroleptic use. Risk factors for tardive dyskinesia include longterm treatment with dopamine receptor antagonists (typical neuroleptics), female sex, presence of a mood disorder, or increasing age. Serotonin-dopamine antagonists, also known as atypical antipsychotics, are associated with a lower risk of development of tardive dyskinesia than typical antipsychotics, so substituting a serotonin-dopamine antagonist might help to limit the abnormal movements without worsening the progression of tardive dyskinesia. Addition of an anticholinergic drug (**choice A**) has not been found to be beneficial in most cases, because tardive dyskinesia does not appear to result directly from disruption of the nigrostriatal dopamine pathway. Although addition of a dopamine receptor antagonist (**choice B**) can suppress tardive dyskinesia temporarily, it can cause a rebound effect of more intense tardive dyskinesia at a later time and is not recommended. Making no changes (**choice C**) is contraindicated, as tardive dyskinesia will continue to progress on the same medication. Stopping the patient’s antipsychotic medication (**choice D**) usually makes tardive dyskinesia worse.
35
A 27-year-old woman, after being examined by her psychiatrist to rule out other medical conditions, has been given a diagnosis of panic disorder with agoraphobia. She currently has no known drug allergies, is taking no medications, and denies any illicit drug use. Given her present diagnosis, which of the following is the most appropriate medication to prescribe? (A) Diazepam (B) Haloperidol (C) Lithium carbonate (D) Paroxetine (E) Temazepam
Respuesta: D The correct **answer is D**. Panic disorder with agoraphobia is a disorder characterized by recurrent unexpected panic attacks with associated agoraphobia (fear of social situations). The current treatment of choice for this disorder is an antidepressant, usually a selective serotonin reuptake inhibitor (SSRI) such as paroxetine, although tricyclic antidepressants (TCAs) and monoamine oxidase inhibitors (MAOIs) have also been demonstrated to have efficacy. Diazepam (**choice A**) and temazepam (**choice E**) are benzodiazepines, which must be used in higher than typical doses in treating panic disorder. Haloperidol (**choice B**) is an antipsychotic and is not indicated in panic disorder. Lithium carbonate (**choice C**) is a treatment for mania and bipolar disorder but is not indicated in panic disorder with agoraphobia.
36
A 45-year-old, homeless Caucasian man is brought into the emergency department by EMS after having been found stuporous in a park. He is known by the emergency department staff to be alcohol dependent, as he has been treated on several occasions for alcohol withdrawal seizures. He is awake but somnolent and complains of abdominal pain and hazy vision. Examination is remarkable for mild bilateral papilledema and generalized abdominal tenderness. Significant laboratory studies suggest an anion gap acidosis. Which of the following is the most appropriate therapy for this patient? (A) Activated charcoal (B) Disulfiram (C) Ethanol (D) Flumazenil (E) Naloxone
Respuesta: C The correct answer is C. This case illustrates poisoning by methanol. A history of known alcohol dependence (such individuals may use methanol as a substitute for ethanol), visual disturbances, and examination findings of altered mental status, papilledema, and abdominal pain suggest the diagnosis. Methanol is converted to formaldehyde by alcohol dehydrogenase, and formaldehyde is further metabolized to formate by aldehyde dehydrogenase. Methanol and ethanol compete for the enzyme alcohol dehydrogenase; the preference of this enzyme for metabolizing ethanol forms the basis for its use in methanol poisoning. Activated charcoal (**choice A**) is used for decontamination of several toxic substances by adsorption to the toxin in the gastrointestinal tract. Activated charcoal has not been shown to adsorb methanol; therefore, it is not useful in methanol poisoning. Disulfiram (**choice B**) is an aldehyde dehydrogenase inhibitor. It is used in alcohol dependence as a deterrent to alcohol consumption. Inhibition of aldehyde dehydrogenase by disulfiram causes an increased concentration of acetaldehyde when ethanol is consumed, producing unpleasant effects. Inhibition of aldehyde dehydrogenase in a person who ingested methanol would lead to accumulation of formaldehyde, which is quite toxic. Flumazenil (**choice D**) is a benzodiazepine receptor antagonist and has no place in the therapy of methanol overdose. Naloxone (**choice E**) is an opioid receptor antagonist and is used in opiate overdose only.
37
A 25-year-old woman is admitted to the psychiatric unit following an overdose of sleeping pills. She seems very angry for being held in the hospital, since she just wanted to “show her boyfriend” (who had broken up with her) how upset she was. She tells the intern on the unit that he is so caring and competent and that she has never met such a doctor. She complains that no one understands her and that the only one who can really help her is the intern. The intern feels flattered, without realizing that the patient is using a defense mechanism. Which of the following defense mechanisms is the patient most likely using? (A) Displacement (B) Distortion (C) Primitive idealization (D) Projection (E) Splitting
Respuesta: C The correct **answer is C**. In primitive idealization, external objects are unrealistically endowed with great power and are either “all good” or “all bad.” All good objects are ideal and omnipotent; the badness of the others is also greatly inflated. Displacement (**choice A**) occurs when an emotion or a drive is shifted from one idea or object to another that resembles the original. This allows the symbolic representation of the original idea in a less threatening way. Distortion (**choice B**) is a defense in which external reality is reshaped to suit inner needs. Unrealistic beliefs, delusions, and hallucinations are used to sustain the feelings of superiority or entitlement. Projection (**choice D**) is a mechanism by which a person attributes his or her intolerable feelings or affects to another person. It is usually seen in psychotic states, especially in paranoid syndromes. Splitting (**choice E**) is a defense in which external objects are divided into “all good” and “all bad,” accompanied by abrupt shifting of an object from one category to another or sudden reversal of feelings about the same person.
38
A 38-year-old African American woman visits a psychiatrist for recurrent depression. She states that she has been taking medication for depression for several years, but is now interested in starting St. John’s wort for her current episode. Her psychiatrist conducts a literature search and discovers an article about the use of St. John’s wort for depression. The article describes six randomized, placebocontrolled trials comparing St. John’s wort with placebo; the data obtained from these studies were pooled to generate new data. Which of the following research designs was used in the study described? (A) Case-control (B) Case report (C) Cohort (D) Cross-sectional survey (E) Meta-analysis
Respuesta: E The correct **answer is E**. A meta-analysis makes use of data from prior studies to synthesize new data and determine more precise statistical information for a particular question. The article described pools the data from six studies concerned with the use of St. John’s wort in depression to make broader generalizations than could be made from a single study. Case-control studies (**choice A**) compare patients with a particular disease or condition with individuals (“controls”) who have similar demographics but do not have the disease in question. Case-controls do not pool data from other studies as meta-analyses do. A case report (**choice B**) describes the medical history of one particular patient. In cohort studies (**choice C**), two or more groups of individuals with similar characteristics, who differ on the basis of exposure to a particular agent, are followed over time to determine how many individuals in each of the groups develop a particular disease. In cross-sectional surveys (**choice D**), data are collected at a given time-point but may refer to past medical history. As in the other incorrect choices, these data are used to produce a single study, which may be used in a meta-analysis at some later point.
39
A 62-year-old man with a diagnosis of schizophrenia, paranoid type, has been stable on medication for years. Prior to that, he had several severe exacerbations that led to prolonged inpatient treatment of several months’ duration. During an appointment with his psychiatrist, he complains of declining vision. He is referred to an ophthalmologist, who diagnoses pigmentary retinopathy. Which of the following medications would most likely have caused this complication? (A) Clozapine (B) Haloperidol (C) Pimozide (D) Thioridazine (E) Thiothixene
Respuesta: D The correct **answer is D**. Thioridazine belongs to the phenothiazine group of antipsychotics that can cause pigmentary retinopathy when doses higher than recommended are given. Clinically, the patient experiences diminished visual acuity, brownish coloring of vision, and impaired night vision. Funduscopic examination discloses the deposits of pigment. The condition is listed as a psychiatric emergency. Remaining within recommended dose limits could reduce the risk of complications. Clozapine (**choice A**) is a dibenzodiazepine derivative that can cause diverse side effects, such as agranulocytosis, orthostatic hypotension, hepatitis, fever, seizures, pulmonary embolism, and hyperglycemia. The most common gastrointestinal side effect is constipation. Hypersalivation is also very common. Haloperidol (**choice B**) belongs to butyrophenone group and is a potent antipsychotic. Its side effects are related to dopaminergic D2 blockade, but it can also cause cardiovascular and gastrointestinal side effects and autonomic reactions. Cataracts, visual disturbances, and retinopathy have been very rarely described. Pimozide (**choice C**) belongs to diphenylbutylpiperidine group. It can cause tardive dyskinesia and neuroleptic malignant syndrome like other antipsychotics, but most commonly it causes sedation and drowsiness, gastrointestinal symptoms, and ECG changes with prolongation of the QT interval. It can rarely cause blurred vision and cataracts. Thiothixene (**choice E**) is a thioxanthene derivative that is effective in the management of psychotic disorders. Adverse reactions involving the CNS (drowsiness, extrapyramidal symptoms), cardiovascular system (hypotension, tachycardia), and liver (elevated hepatic enzymes) have been reported. Other side effects include the ones usually produced by other phenothiazines; pigmentary retinopathy is not one of them.
40
A 71-year-old Caucasian man is brought to the physician by his wife for evaluation. The patient has a history of Parkinson disease, diagnosed by another physician 6 years ago, for which he takes Ldopa/carbidopa 4 times a day. His medical history is also significant for congestive heart failure, which is treated with digoxin and metoprolol, and depression, which is treated with fluoxetine. The man describes his mood as generally good, but he is particularly disturbed by visual hallucinations described as “people and small animals” that he is certain are not there and by thoughts that his wife is trying to kill him. These symptoms have been intermittently present for several months but are now getting much worse. He has not been to his internist for more than 5 years, but he assures you he has been taking his medications exactly as prescribed. Which of the following is the most likely explanation for his delusions? (A) Delusional disorder (B) Depression with psychotic features (C) Digoxin toxicity (D) L-dopa/carbidopa toxicity (E) Schizophrenia
Respuesta: D The correct **answer is D**. Sinemet, which is a combination drug composed of L-dopa and carbidopa, is not infrequently implicated in emergence of psychotic symptoms in patients with advanced Parkinson disease who require frequent dosing. Increased load of Ldopa causes overactivity of dopaminergic cells in the mesolimbic and mesocortical tracts, which are believed to be involved in producing psychotic symptoms. In delusional disorder (**choice A**), hallucinations are absent. Although depression with psychotic features (**choice B**) may be associated with visual hallucinations and delusions, this man describes his mood as “generally okay.”. Digoxin toxicity (**choice C**) may affect vision; however, it is not associated with frank psychosis. New-onset schizophrenia (**choice E**) would be exceedingly rare in a 71-year-old man.
41
A 45-year-old Asian American female nurse is brought to the emergency department by ambulance after having been discovered unresponsive by her husband 30 minutes earlier. En route to the hospital, paramedics determine her serum glucose to be 13 mg/dL. After infusion of dextrose, the patient regains consciousness. On interview, the patient claims that she has a long history of severe hypoglycemia that often requires visits to the emergency department and, occasionally, admission to the hospital. Review of the chart reveals 10 such visits in the past year. Repeated work-ups for hypoglycemia have been negative. Questioning of her husband reveals that these episodes began to occur after the death of the patient’s mother from cancer. Physical examination and routine laboratory tests are unremarkable. Which of the following is the most likely diagnosis? (A) Factitious disorder (B) Ganser syndrome (C) Hypochondriasis (D) Malingering (E) Anorexia nervosa
Respuesta: A The correct **answer is A**. In factitious disorder, the patient intentionally produces or feigns signs or symptoms, although the motivation behind such action is largely unconscious and often serves to place the patient into the sick role. In this case, the patient may be surreptitiously injecting insulin or taking oral hypoglycemics. It is not uncommon for the affected individual to be a member of the health care community who may be familiar with the presentation of certain disease states. Ganser syndrome (**choice B**) is perhaps best classified as a subtype of malingering, commonly seen in prison inmates, and is characterized by the use of approximate, but completely inappropriate, answers. For example, when asked what the middle color of a traffic light is, one might reply “blue.” The deception is advertent in an attempt to minimize criminal culpability by feigning mental illness. Hypochondriasis (**choice C**) is the preoccupation with fears of having a serious physical illness based on a person’s misinterpretation of bodily symptoms. In this vignette, the patient intentionally creates a physical disease within herself so as to unconsciously achieve secondary gain. In hypochondriasis, there is no apparent unconscious motivation for believing to be ill. Malingering (**choice D**) is the voluntary production of physical or psychological symptoms for external gain, such as to attain admission to the hospital for the sole purpose of shelter or to make a workers’ compensation claim. The difference between malingering and factitious disorder is that, in malingering, both the production of and motivation behind displaying symptoms of illness are conscious. In anorexia nervosa (**choice E**), there is an intense fear of gaining weight, and weight reduction is achieved through either severely restricting dietary intake or binge eating followed by purging. Intentional production of hypoglycemia would not be consistent with a diagnosis of anorexia nervosa.
42
A 39-year-old Caucasian woman visits her primary care physician for a routine examination. She denies any physical or psychiatric complaints, and her physical examination is within normal limits. Her physician advises her of the importance of eating a well- balanced diet and exercising on a regular basis. He then asks whether she has any questions before he sees his next patient. She suddenly becomes tearful and says, “Actually, yes. My 17-year-old son just told me he is gay, and I don’t know what to do. I’ve always been taught that homosexuality is wrong, but he is my son and I love him. What should I do?” Which of the following would be the most appropriate response? (A) “Experimenting with homosexuality is completely normal behavior in adolescents. I’m sure this is a phase that will pass with time.” (B) “Homosexuality is considered a normal variant of human sexuality. However, I can see that you are concerned. Let’s talk about what’s on your mind.” (C) “These things need to be discussed with a psychiatrist. I can make a referral.” (D) “Your son’s sexual life is his private business and he needs to discuss that with his own physician.” (E) “You seem rather upset. What is it that you don’t like about gay people?”
Respuesta: B The correct **answer is B**. Sexuality, in general, and sexual preference, in particular, are subjects that many physicians are uncomfortable addressing; however, they are important aspects of human life and have medical and psychosocial ramifications. Although homosexuality is no longer considered a mental illness, the discovery of one’s own sexual preference may cause significant distress in that person and in other family members. Accordingly, physicians are charged with the responsibility to identify these concerns and address them with any involved. Although experimentation with homosexual behavior, particularly in adolescence (**choice A**), is much more common than lifelong homosexuality, it would be inappropriate to assume that any such behavior is mere experimentation. Disseminating such information would likely serve to mitigate the discomfort felt by both the physician and the patient in this case; however, it would be misleading. Any physician should feel comfortable discussing issues of sexuality, not just psychiatrists (**choice C**). Such disclosure should be treated with the same degree of sensitivity and confidentiality as any other personal information. Although the patient’s son’s private affairs are his own business, it would not be considered a breach of confidentiality to discuss the content matter that the patient has disclosed (compare with **choice D**). In fact, refusing to discuss the issue, and thereby dismissing the patient’s distress, would be considered inappropriate. Although it is appropriate to acknowledge a patient’s feelings, it is not acceptable to make assumptions as to why she may feel upset; therefore, **choice E** is incorrect. At no point did the patient state she did not like gay people, and it is wrong for the physician to assume such
43
A 12-year-old boy has been sent from his school to a psychiatrist for evaluation. During the past several months, the teacher has observed that the boy has been restless and fidgety. He repeatedly clears his throat and needs to spit. Recently, he distracted the whole class by making loud, barking noises. The parents have observed that he sometimes has facial tics and blinking, and shakes his head on one side. He has been otherwise doing well academically and is very neat. Which of the following is the most likely diagnosis? (A) Attention deficit/hyperactivity disorder (B) Autistic disorder (C) Huntington disease (D) Oppositional defiant disorder (E) Tourette syndrome
Respuesta: E The correct **answer is E**. Tourette syndrome belongs to the group of tic disorders with onset before age 18. It is most common in boys. Multiple motor and vocal tics are present many times a day, every day, for more than a year, with no tic-free period greater than 3 months. Coprolalia and echolalia occur later. Attention deficit/hyperactivity disorder (**choice A**) is more frequently seen in males and is characterized by inattention, hyperactivity, or impulsivity persisting at least 6 months. It is not due to any pervasive developmental disorder or other mental disorder. Autistic disorder (**choice B**) is a pervasive developmental disorder starting in early childhood. It is characterized by qualitative impairment in social interaction and communication, as well as restricted repetitive and stereotyped patterns of behavior or interests. Delay or abnormal functioning with onset prior to 3 years is seen in the areas of social interaction, language, and communication or symbolic play. Huntington disease (**choice C**) is a dominantly inherited disorder that begins in mid-adult life and is characterized by a hyperkinetic movement disorder and intellectual decline. Involuntary choreiform movements typically affect the limbs or trunk. Intellectual decline usually progresses into dementia of the subcortical type. Oppositional defiant disorder (**choice D**) occurs in childhood and consists of a pattern of negativistic, hostile, and defiant behavior lasting at least 6 months. During this period, at least four of the symptoms must be present. These include loss of temper, arguments with adults, blaming others for own mistakes, deliberately annoying others, and refusing to comply with rules.
44
A 61-year-old woman has slowly been developing movement abnormalities. She finds herself moving more slowly and stiffly and she notices a tremor in her hands that was not there a year ago. Her husband thinks that she is slower when she starts to move but has trouble slowing down after she gets going. Once she is moving, her steps seem more like a shuffle than a walk. He also thinks that her facial expressions seem blunted, that she does not blink as much, and that the tone of her speech has become more monotonous. This woman is at the highest risk of having which of the following psychiatric disturbances? (A) Anxiety (B) Auditory hallucinations (C) Dementia (D) Depression (E) Insomnia
Respuesta: D The correct **answer is D**. This woman has Parkinson disease (PD), which is an idiopathic subcortical degenerative disease that predominantly affects cells containing dopamine. The typical age of onset is 50-60 years, and the clinical course is chronic and progressive, with severe disability usually after 10 years of illness. Subcortical diseases in general affect the “three M’s": movement, mentation, and mood. In PD, all three are affected but not necessarily uniformly. The movement abnormalities are prototypically tremor, rigidity, and bradykinesia. Disorders of mentation or cognition affect people in various ways. Most patients complain of slowed thinking, and about 20 to 30% are found to have dementia, with a higher likelihood in those with late-onset disease (i.e., after age 70). Mood disorders are common in PD, with the mean frequency of depression reported at 40%. No relation has been shown to exist between the frequency and severity of depression and the patient’s current age, age of onset, or severity of PD symptoms and response to medication. It is postulated that depression is a primary manifestation of brain deterioration and not a reactive psychologic response to disability and chronic illness. There is no known increased incidence of anxiety (**choice A**) among PD patients when compared with the general age-matched population. Psychosis as part of PD has been reported in the context of mood disorders, for example in psychotic depression, but this is not particularly common. Visual, but not auditory (**choice B**), hallucinations are a neuropsychiatric feature of dementia with Lewy bodies. This is a type of dementia related to the presence of Lewy bodies in the brainstem and cerebral cortex. It is characterized by a progressive course, visual hallucinations, delusions, fluctuating attention, difficulty with executive functioning, and cognitive deficits. Mood changes are common as well. As stated above, about 20 to 30% of patients with PD are found to have dementia (**choice C**), with a higher likelihood in those with late-onset disease (i.e., after age 70). About 40% of nondemented PD patients demonstrate some neuropsychological impairment, mostly in visuospatial abilities. Insomnia (**choice E**) is not characteristic of PD. Often, if a patient with PD does have sleep disturbances, it is in the context of depressive symptomatology.
45
A patient taking haloperidol for the first time experiences an involuntary twisting of her neck. She is given benztropine for relief. This medication may cause which of the following side effects? (A) Diaphoresis (B) Diarrhea (C) Fevers (D) Insomnia (E) Mydriasis
Respuesta: E The correct **answer is E**. Benztropine is an antimuscarinic agent given to relieve symptoms of acute dystonic reactions. Blockade of cholinergic transmission produces blurred vision, confusion, constipation, dry skin, hallucinations, mydriasis, and urinary retention. Diaphoresis (**choice A**) and diarrhea (**choice B**) occur with cholinergic, not anticholinergic, drugs. Fevers (**choice C**) and insomnia (**choice D**) are not side effects observed with either cholinergic or anticholinergic drugs.
46
A 31-year-old woman has refused to leave her home for the past 4 months. She had been on a crowded subway train when she felt lightheaded and nervous, had a headache, and felt like she could not breathe. She felt tingling in her left arm and left side of her head. Since then, she has been afraid of going out on her own, worried that another attack would happen and no one would help her. She has no prior medical history, and the attacks do not occur at home. Which of the following is the most likely diagnosis? (A) Agoraphobia (B) Avoidant personality disorder (C) Dependent personality disorder (D) Somatization disorder (E) Specific phobia
Respuesta: A The correct **answer is A**. Agoraphobia is a fear of being in public places, away from home or without company. It refers to the fear of being in a situation or place from which there is no easy escape, such as in a theater, on public transportation, or in stores. The symptoms are usually the same as the ones seen in panic attacks, developing abruptly and lasting several minutes. Some of the symptoms include palpitations, sweating, paresthesias, dizziness, feeling of choking, trembling, chest pain, derealization, chills, and fear of losing control or dying. Avoidant personality disorder (**choice B**) is a pervasive pattern of social inhibition and hypersensitivity to negative evaluation beginning in early adulthood. It is characterized by such symptoms as inhibition in new interpersonal situations, viewing oneself as inept, preoccupation with being criticized, and unwillingness to get involved with people unless sure of being liked. Dependent personality disorder (**choice C**) is a pervasive need to be taken care of that leads to clinging behavior. At least five symptoms, including difficulty expressing disagreement, difficulty making decisions, feelings of helplessness when alone, fear of taking responsibility for major areas in life, and fear of being left alone to take care of oneself, are typically present. Somatization disorder (**choice D**) begins before age 30. Typically, there is significant impairment present in major areas of functioning secondary to symptoms. Symptoms include four pain symptoms: one pseudoneurologic, one sexual, and two gastrointestinal. Specific phobia (**choice E**) is marked, excessive fear that is cued by the presence or anticipation of a specific object or situation. Exposure to the feared object or situation results in an anxiety response, and the person is aware that the fear is unreasonable. The symptoms can cause significant impairment in everyday functioning.
47
A 29-year-old Asian American woman is seen for the first time by a psychiatrist. She complains of depressed mood, decreased appetite, lack of interest in her usual social activities, decreased energy, and a difficult time concentrating at work for the past 3 months. Occasionally, she feels too fatigued to go to work. Although she denies thoughts of wanting to kill herself, she claims that sometimes she feels that she would be “better off dead.” The psychiatrist prescribes medication for her and asks her to return in 6 weeks. On the return visit, she reports that her symptoms are significantly improved. If this patient did not improve and she reported other complaints, such as dry skin, constipation, intolerance to cold temperatures, and hoarse voice, which of the following diagnostic tests would be most appropriate? (A) Follicle stimulating hormone (FSH) level (B) Thyroid stimulating hormone (TSH) level (C) Urinalysis (D) Urinary hCG level (E) Venereal Disease Research Laboratory (VDRL) test
Respuesta: B The correct **answer is B**. The differential diagnosis of major depression includes hypothyroidism; a TSH level is a good screening test for this condition. This question could have been answered independently of the clinical vignette because the classic symptoms of hypothyroidism are present. Measurement of follicle stimulating hormone levels (**choice A**) has utility in psychiatry in perimenopausal women to rule out depression secondary to menopause. The patient’s young age and presence of symptoms suggestive of hypothyroidism would make this test inappropriate. A urinalysis (**choice C**) is performed to rule out a urinary tract infection, which may be responsible for delirium in certain predisposed patients (elderly, demented, medically compromised). The above patient’s symptoms, however, are strongly suggestive of hypothyroidism. A urine hCG test (**choice D**) would be helpful in diagnosing pregnancy, not hypothyroidism. The Venereal Disease Research Laboratory (VDRL) test (**choice E**) is used to screen for primary and secondary syphilis. Neurosyphilis, characterized by personality changes, irritability, and psychosis, is diagnosed by performing a VDRL on the CSF.
48
A 38-year-old woman is brought to the emergency department by her husband because of her “odd” behavior over the past day. She seems confused and is unable to recall her name, address, birthday, or age. She is also unable to recall anything about her personal history from the previous 2 months. She and her husband deny any stressful life events or trauma that may have contributed to this. The patient appears appropriately concerned and her mood and affect are congruent with her thought content. Her speech is normal and her thought is organized and goal directed. She denies any use of overthe-counter medication, alcohol, or drugs. Her husband confirms all of the information that she provides. Physical and neurologic examination, blood chemistry, urine toxicology, MRI of the head, lumbar puncture, and EEG are all within normal limits. This patient is most likely suffering from which of the following conditions? (A) Anterograde amnesia (B) Confabulation (C) Dissociative amnesia (D) Hypermnesia (E) Transient global amnesia
Respuesta: C The correct **answer is C**. Once the differential diagnosis of acute memory loss is done, and delirium, dementia, trauma, and substance use are ruled out, the diagnosis of dissociative amnesia can be established. It usually has a circumscribed nature, and the patient’s perplexity, confusion, and amnesia following traumatic events last for a discrete period of time. It is characterized by the inability to recall personal identification that is deeply embedded information and that is only destroyed in very advanced stages of Alzheimer disease. Anterograde amnesia (**choice A**) is defined by a specific short-term memory deficit in which patients are unable to recall new information or events that happened in the previous minutes before the blackout. During the blackout, remote memory is intact, as is ability to perform tasks. This can be caused by alcohol, medications, drugs, or trauma. Confabulation (**choice B**) can be seen in chronic alcoholics with dementia, and is characterized by gaps in memory that are filled with events that never happened. Hypermnesia (**choice D**) describes an exaggerated degree of retention and recall. Transitory global amnesia (**choice E**) is characterized by the inability to remember new information, such as date and location. Personal information is, however, retained. This discrepancy separates it from dissociative amnesia. Electroencephalogram can show spikes. The cause may be a vascular abnormality that leads to temporary ischemia.
49
A 22-year-old Native American woman sees a psychiatrist for the first time shortly following the dissolution of a 3-year relationship with her boyfriend. She states that she is intensely upset and depressed and experiences frequent crying spells. For the past few days, she has found it difficult to go to work because she cannot seem to shift her focus away from the breakup. Although she can usually find comfort from talking things over with her mother, she notes that she seems to direct her anger at her mother and she feels guilty about this. Which of the following psychotherapeutic interventions is most appropriate? (A) Cognitive-behavioral therapy (B) Hypnosis (C) Psychoanalysis (D) Social skills training (E) Supportive psychotherapy
Respuesta: E The correct **answer is E**. Supportive psychotherapy is used to help patients through difficult situations. It may incorporate the philosophies underlying other types of insightoriented psychotherapy, but the main goal is to show sympathy, concern, and interest. When the current crisis has passed, other types of therapies may be appropriate. Cognitive-behavioral therapy (CBT) (**choice A**) is concerned with repairing cognitive distortions and changing maladaptive behaviors. It may be useful in conditions such as dysthymia or anxiety disorders. Hypnosis (**choice B**) may be helpful in dissociative and conversion disorders. An adjustment disorder, such as that described above, is most amenable to a supportive approach. Psychoanalysis (**choice C**) is relatively contraindicated in this case. In psychoanalysis, the therapist maintains a neutral attitude; doing so in this situation may be perceived as rejection and would serve to alienate the patient, not comfort her. Social skills training (**choice D**) focuses on developing abilities in relating to others and is reserved for patients with severe mental illness, such as schizophrenia.
50
A 37-year-old man is brought to the emergency department by police after he was found wandering around naked and shouting obscenities. The patient is approached by a female nurse and becomes very agitated. He begins to shout at her and threatens to kill her. He is given an intramuscular injection of haloperidol. Ten minutes later, he complains that his tongue feels thick and that he has difficulty moving because his muscles are stiff. Vital signs are stable. Breathing is unlabored and physical examination is remarkable for diffuse muscular rigidity. Which of the following medications would effectively treat this condition? (A) Bromocriptine (B) Carbidopa/L-dopa (C) Carisoprodol (D) Diphenhydramine
Respuesta: D The correct **answer is D**. The patient is experiencing an acute dystonic reaction as a result of the extrapyramidal side (EPS) effects of antipsychotic medication. This condition is best treated with injectable diphenhydramine. Bromocriptine (**choice A**) is a dopamine agonist that can be used in neuroleptic malignant syndrome (NMS), a condition associated with antipsychotic use and characterized by autonomic instability, altered mental status, and muscular rigidity. Carbidopa/L-dopa (Sinemet; **choice B**) is used in idiopathic parkinsonism (Parkinson disease). It has no place in the treatment of EPS. Carisoprodol (Soma; **choice C**) is a muscle relaxant often used for acute or chronic musculoskeletal conditions and ankylosing spondylitis. EPS is treated with an agent that has prominent anticholinergic or antihistaminergic properties.